15
$\begingroup$

Prove, if possible in an elementary way, that $\sum_{n=1}^{\infty}\frac{1}{p_n(p_{n+1}-p_n)}$ converges/diverges, where $p_n$ denotes the $n^{\textrm{th}}$ prime.

$\endgroup$
7
  • 13
    $\begingroup$ Ummm... people don't like it when you use the imperative voice "Prove X". People do like it if you give context for the problem - how did it arise? etc. $\endgroup$ Feb 4, 2016 at 4:06
  • 2
    $\begingroup$ I essentially stumbled on this problem by accident. It seems interesting primarily as any "trivial" elementary bounds appear to fail, but using more machinery like Brun's estimates appears non-trivial. $\endgroup$
    – mssmath
    Feb 4, 2016 at 4:10
  • 1
    $\begingroup$ The converges/diverges refers to proving that it either converges OR it diverges. Still point well taken for the future. $\endgroup$
    – mssmath
    Feb 4, 2016 at 4:11
  • 17
    $\begingroup$ The convergence of your series follows from upper bound sieve estimates. In fact, a somewhat stronger result is shown in this paper: Erdös, Paul(H-AOS); Nathanson, Melvyn B.(1-CUNY7) On the sum of the reciprocals of the differences between consecutive primes. Number theory (New York, 1991–1995), 97–101, Springer, New York, 1996. $\endgroup$ Feb 4, 2016 at 4:23
  • 2
    $\begingroup$ I was about to say that the series "should" converge using crude probabilistic heuristics: If $p_n$ is a prime, the next integers should have something like a $1/\log p_n$ probability of being prime. So the expected value of $1/(p_{n+1}-p_n)$ given $p_n$ should be something like $\log\log p_n/\log p_n$. Now the sum is something like $\sum \frac{\log\log n}{n\log^2n}$. Of course now I should check how this checks out in the Erdös-Nathanson paper. $\endgroup$ Feb 4, 2016 at 4:28

1 Answer 1

23
$\begingroup$

By popular demand, I am converting my comment above to an answer. Yes, the series converges. In the paper

Erdös, Paul(H-AOS); Nathanson, Melvyn B.(1-CUNY7) On the sum of the reciprocals of the differences between consecutive primes. Number theory (New York, 1991–1995), 97–101, Springer, New York, 1996

the authors show that $$\sum_{n=3}^{\infty} \frac{1}{n(\log\log{n})^c (p_{n+1}-p_n)}$$ converges for every choice of $c>2$. (I start the sum at $n=3$ rather than $n=2$ as as is done in the paper to avoid the annoyance that $\log\log{2} < 0$.) They also present a heuristic argument that the series diverges when $c=2$. The main tool in the proof is Brun's classical upper bound sieve estimate for the number of prime pairs $p, p+N$ with $p$ below a given bound.

Since $p_n$ is bounded below by a positive constant multiple of $n\log{n}$, the convergence of your series follows by comparison.

$\endgroup$

Your Answer

By clicking “Post Your Answer”, you agree to our terms of service and acknowledge you have read our privacy policy.

Not the answer you're looking for? Browse other questions tagged or ask your own question.